Một số bất đẳng thức hình học

Mục lục 1 Mở đầu . 3 Chương 1. Các bất đẳng thức trong tam giác và tù giác 6 1.1. Các bất đẳng thức đại số cơ bản . 6 1.2. Các đẳng thức và bất đẳng thức cơ bản trong tam giác 8 1.2.1. Các đẳng thức cơ bản trong tam giác . 8 1.2.2. Các bất đẳng thức cơ bản trong tam giác . 10 1.3. Bất đẳng thức trong tam giác 11 1.3.1. Bất đẳng thức về độ dài các cạnh . 11 1.3.2. Bất đẳng thức về các đại lượng đặc biệt 14 1.4. Các bất đẳng thức sinh ra từ các công thức hình học 17 1.5. Bất đẳng thức trong các tam giác đặc biệt 23 1.5.1. Các bất đẳng thức trong tam giác đều 23 1.5.2. Các bất đẳng thức trong tam giác vuông và tam giác cân 27 1.6. Các bất đẳng thức khác trong tam giác . 29 1.7. Các bất đẳng thức trong tù giác . 40 1.7.1. Các bất đẳng thức cơ bản trong tù giác . 41 1.7.2. Các bất đẳng thức khác trong tù giác . 45 Chương 2. Bất đẳng thức Ptolemy và các mở rộng 48 2.1. Đành lí Ptolemy . 48 2.2. Bất đẳng thức Ptolemy . 53 2.3. Đành lí Bretschneider . 63 2.4. Đành lí Casey 63 2.5. Mở rộng bất đẳng thức Ptolemy trong không gian . 68 Chương 3. Bất đẳng thức Erdos-Mordell và các mở rộng 70 3.1. Bất đẳng thức Erdos-Mordell trong tam giác . 70 3.2. Bất đẳng thức Erdos-Mordell trong tam giác mở rộng 79 3.3. Mở rộng bất đẳng thức Erdos-Mordell trong tù giác . 85 3.4. Mở rộng bất đẳng thức Erdos-Mordell trong đa giác . 87 3.5. Mở rộng bất đẳng thức Erdos-Mordell trong tứ diện . 90 Chương 4. Các bất đẳng thức có trọng 92 4.1. Bất đẳng thức dạng Hayashi và các hệ quả . 92 4.1.1. Bất đẳng thức Hayashi . 92 4.1.2. Các hệ quả của bất đẳng thức hyashi . 94 4.1.3. Bài toán áp dụng 94 4.2. Bất đẳng thức Weizenbock suy rộng và các hệ quả 96 4.2.1. Bất đẳng thức Weizenbock suy rëng 96 4.2.2. Các hệ quả của bất đẳng thức Weizenbock suy rộng 101 4.3. Bất đẳng thức Klamkin và các hệ quả 105 4.3.1. Bất đẳng thức Klamkin 105 4.3.2. Các hệ quảcõa bất đẳng thức Klamkin . 106 4.4. Bất đẳng thức Jian Liu và các hệ quả 108 4.4.1. Bất đẳng thức Jian Liu . 108 4.4.2. Các hệ quả của bất đẳng thức Jian Liu . 110 Kết luận . 116 Tài liệu tham khảo . 117

pdf120 trang | Chia sẻ: lvcdongnoi | Lượt xem: 5071 | Lượt tải: 2download
Bạn đang xem trước 20 trang tài liệu Một số bất đẳng thức hình học, để xem tài liệu hoàn chỉnh bạn click vào nút DOWNLOAD ở trên
2 + (x1 + x2) y3 ≥ 2 √ (x1x2 + x2x3 + x3x1) (y1y2 + y2y3 + y3y1) Dấu đẳng thức xảy ra khi và chỉ khi x1 y1 = x2 y2 = x3 y3 . Chứng minh. Áp dụng bất đẳng thức Cauchy-Schwarz ta được (x2 + x3) y1 + (x3 + x1) y2 + (x1 + x2) y3 =(x1 + x2 + x3) (y1 + y2 + y3)− (x1y1 + x2y2 + x3y3) = √[ x21 + x 2 2 + x 2 3 + 2 (x1x2 + x2x3 + x3x1) ] [ y21 + y 2 2 + y 2 3 + 2 (y1y2 + y2y3 + y3y1) ] − (x1y1 + x2y2 + x3y3) ≥2 √ (y1y2 + y2y3 + y3y1) (x1x2 + x2x3 + x3x1) + √( x21 + x 2 2 + x 2 3 ) ( y21 + y 2 2 + y 2 3 ) − (x1y1 + x2y2 + x3y3) ≥2 √ (y1y2 + y2y3 + y3y1) (x1x2 + x2x3 + x3x1). Bổ đề 3.3. Cho tam giác ABC và x, y, z là các số thực dương. Bất đẳng thức sau luôn đúng x sinA+ y sinB + z sinC ≤ 1 2 (xy + yz + zx) √ x+ y + z xyz . (3.19) Đẳng thức xảy ra khi và chỉ khi x = y = z và ABC là tam giác đều. Chứng minh. Trước hết ta có x2 + y2 + z2 + 2yz cos 2A+ 2zx cos 2B + 2xy cos 2C ≥ 0. (3.20) Vì x2 + y2 + z2 + 2yz cos 2A + 2zx cos 2B + 2xy cos 2C = (x+ z cos 2B + y cos 2C)2 + (z sin 2B − y sin 2C)2 ≥ 0. Trong (3.20) thay (x, y, z) bằng ( 1 x , 1 y , 1 z ) , ta được bất đẳng thức x cos 2A+ y cos 2B + z cos 2C ≥ −1 2 ( yz x + zx y + xy z ) . (3.21) www.MATHVN.com - HOANG NGOC QUANG, Yen Bai 82 Thay cos 2A = 1− 2 sin2A, cos 2B = 1− 2 sin2B, cos 2C = 1− 2 sin2C vào (3.21), ta được 2 ( xsin2A+ ysin2B + zsin2C ) ≤ x+ y + z + 1 2 ( yz x + zx y + xy z ) hay xsin2A+ ysin2B + zsin2C ≤ (yz + zx+ xy) 2 4xyz . (3.22) Bây giờ, áp dụng bất đẳng thức Cauchy-Schwarz và sử dụng (3.22), ta được (x sinA+ y sinB + z sinC)2 ≤ (xsin2A+ ysin2B + zsin2C) (x+ y + z) ≤(yz + zx+ xy) 2 4xyz (x+ y + z) . Suy ra x sinA+ y sinB + z sinC ≤ 1 2 (yz + zx + xy) √ x+ y + z xyz . Đẳng thức xảy ra khi và chỉ khi x+ z cos 2B + y cos 2C = 0 z sin 2B − y sin 2C = 0 sinA = sinB = sinC ⇔ { x = y = z A = B = C ⇔ { x = y = z ∆ABC đều. Bổ đề 3.4. Cho tam giác ABC và điểm P tùy ý trong nó, ta luôn có bất đẳng thức√ abr1r2 + bcr2r3 + car3r1 ≥ 2 (r1r2 + r2r3 + r3r1) . (3.23) Chứng minh. Đặt x = [PBC] , y = [PCA] , z = [PAB], ta có x x+ y + z = [PBC] [ABC] = r1a bc sinA = 2r1 bc . a 2 sinA = 2Rr1 bc . Suy ra r1 = bc 2R . x x+ y + z . Tương tự r2 = ca 2R . y x+ y + z , r3 = ab 2R . z x+ y + z . Do đó bất đẳng thức (3.23) tương đương với abc 2R(x+ y + z) √ xy + yz + zx ≥ abc R(x+ y + z)2 ( a 2R yz + b 2R zx+ c 2R xy ) www.MATHVN.com - HOANG NGOC QUANG, Yen Bai 83 hay 1 2 (x+ y + z) √ xy + yz + zx ≥ yz sinA+ zx sinB + xy sinC. (3.24) Bất đẳng thức (3.24) suy ra từ bổ đề 3.3 bằng cách thay (x, y, z) bằng( 1 x , 1 y , 1 z ) . Hoàn thành chứng minh bổ đề 3.4. Định lý 3.4. Cho P,Q là hai điểm tùy ý nằm trong tam giác ABC. Gọi R1, R2, R3 và r1, r2, r3 lần lượt là các khoảng cách từ P tới các đỉnh A,B,C và các cạnh BC,CA,AB. Đối với điểm Q định nghĩa D1, D2, D3 và d1, d2, d3 một cách tương tự. Khi đó ta có các bất đẳng thức sau. R1D1 +R2D2 +R3D3 ≥ 4 (r1r2 + r2r3 + r3r1) , (3.25) R1D1 +R2D2 +R3D3 ≥ 4 (d1d2 + d2d3 + d3d1) . (3.26) Chứng minh. Hình 3.9 Theo các bất đẳng thức (3.2), (3.3), (3.4), bổ đề 3.2, bổ đề 3.4 và định lí 4.1 ta có R1D1 +R2D2 +R3D3 = aR1. D1 a + bR2. D2 b + cR3. D3 c ≥ (cr2 + br3) D1 a + (ar3 + cr1) D2 b + (br1 + ar2) D3 c ≥2 √ (abr1r2 + bcr2r3 + ca3r1) ( D1D2 ab + D2D3 bc + D3D1 ca ) ≥2 √ abr1r2 + bcr2r3 + car3r1 ≥4 (r1r2 + r2r3 + r3r1) . www.MATHVN.com - HOANG NGOC QUANG, Yen Bai 84 Bất đẳng thức (3.25) được chứng minh. Còn bất đẳng thức (3.26) được chứng minh tương tự. Hệ quả 3.2. Với mọi tam giác ABC và hai điểm trong nó P,Q, ta có R1D1 +R2D2 +R3D3 ≥ 4 √ (r1r2 + r2r3 + r3r1) (d1d2 + d2d3 + d3d1). (3.27) Chứng minh. Cộng theo vế hai bất đẳng thức (3.25) và (3.26), sau đó áp dụng bất đẳng thức AM −GM , ta thu được bất đẳng thức (3.27). Định lý 3.5. Cho tam giác ABC bất kỳ với độ dài các cạnh là a, b, c và P là điểm nằm trong tam giác. Kí hiệu R1, R2, R3 là khoảng cách từ P tương ứng đến các đỉnh A,B,C và r1, r2, r3 tương ứng là khoảng cách từ P đến các cạnh BC,CA,AB. Với mọi số thực α ∈ [0, 1] ta luôn có Rα1 +R α 2 +R α 3 ≥ 2α (rα1 + rα2 + rα3 ) . Trước hết ta xét bổ đề sau: Bổ đề 3.5. Với mọi số dương x, y và với mọi số thực α thỏa mãn 0 < α ≤ 1 ta có (x+ y)α ≥ 2α−1 (xα + yα) . (3.28) Chứng minh. Ta có bất đẳng thức (3.28) tương đương với( x y + 1 )α ≥ 2α−1 (( x y )α + 1 ) . (3.29) Đặt t = x y > 0, bất đẳng thức (3.29) trở thành (t+ 1)α ≥ 2α−1 (tα + 1) . Xét f(t) = (t+ 1)α− 2α−1 (tα + 1), t > 0. Ta chỉ cần xét với 0 < α < 1 (vì nếu α = 1 thì (3.28) hiển nhiên đúng). Ta có f ′(t) = α (t+ 1) α−1 − αtα−12α−1 = α [ (t+ 1)α−1 − (2t)α−1 ] . f ′(t) = 0⇔ t = 1. Từ bảng xét dấu suy ra f(t) ≥ 0, với mọi t > 0. Vậy bổ đề được chứng minh. Bây giờ ta chứng minh định lý 3.5: Áp dụng (3.2) và bổ đề (3.5) ta có Rα1 ≥ ( r2c a + r3b a )α ≥ 2α−1 [(r2ca )α + (r3ba )α] , www.MATHVN.com - HOANG NGOC QUANG, Yen Bai 85 Hình 3.10 Bảng xét dấu tương tự Rα2 ≥ 2α−1 [( r3a b )α + ( r1c b )α] , Rα3 ≥ 2α−1 [( r1b c )α + ( r2a c )α] . Cộng theo vế ba bất đẳng thức trên, sau đó áp dụng bất đẳng thức AM −GM ta được Rα1 +R α 2 +R α 3 ≥2α−1 [(r1c b )α + ( r1b c )α] + 2α−1 [(r2c a )α + (r2a c )α] + +2α−1 [(r3a b )α + ( r3b a )α] ≥ 2α (rα1 + rα2 + rα3 ) . 3.3. Mở rộng bất đẳng thức Erdos-Mordell trong tứ giác Định lý 3.2 cho ta ý tưởng xây dựng bất đẳng thức Erdos-Mordell mở rộng trong tứ giác Bổ đề 3.6. Giả sử x1, x2, x3, x4, θ1, θ2, θ3, θ4, là các số thực tùy ý và θ1 + θ2 + θ3 + θ4 = pi. Khi đó ta có bất đẳng thức √ 2 (x1x2 cos θ1 + x2x3 cos θ2 + x3x4 cos θ3 + x4x1 cos θ4) ≤ x21 + x22 + x23 + x24. (3.30) Chứng minh. Từ nhận xét θ1+θ2+(θ3 + θ4) = pi và θ3+θ4+(θ1 + θ2) = pi, áp dụng bổ đề 3.1 ta được x1√ 2 x2 cos θ1 + x2 x3√ 2 cos θ2 + x1√ 2 x3√ 2 cos (θ3 + θ4) ≤ 1 2 ( x21 2 + x22 + x23 2 ) , x3√ 2 x4 cos θ3 + x4 x1√ 2 cos θ4 + x3√ 2 x1√ 2 cos (θ1 + θ2) ≤ 1 2 ( x23 2 + x24 + x21 2 ) . Cộng theo vế hai bất đẳng thức trên, với lưu ý rằng cos (θ3 + θ4) + cos (θ1 + θ2) = 0, ta thu được bất đẳng thức cần chứng minh.  www.MATHVN.com - HOANG NGOC QUANG, Yen Bai 86 Định lý 3.6. Tứ giác lồi ABCD, P là một điểm nằm trong nó. Kí hiệu R1, R2, R3, R4 lần lượt là khoảng cách từ P tới các đỉnh A,B,C,D và l1, l2, l3, l4 độ dài các đường phân giác của các góc ÂPB, B̂PC, ĈPD, D̂PA, tương ứng. Khi đó R1 +R2 +R3 +R4 ≥ √ 2 (l1 + l2 + l3 + l4) . (3.31) Chứng minh. Đặt ÂPB = 2α, B̂PC = 2β, ĈPD = 2γ, D̂PA = 2δ, ta có α + β + γ + δ = pi. Hình 3.11 Sử dụng công thức đường phân giác và bất đẳng thức AM −GM ta có l1 = 2R1R2 R1 +R2 cosα ≤ √ R1R2 cosα, l2 = 2R2R3 R2 +R3 cos β ≤ √ R2R3 cos β, l3 = 2R3R4 R3 +R4 cos γ ≤ √ R3R4 cos γ, l4 = 2R4R1 R4 +R1 cos δ ≤ √ R4R1 cos δ. Từ 4 bất đẳng thức trên và áp dụng bổ đề 3.6 ta có √ 2 (l1 + l2 + l3 + l4) ≤ √ 2( √ R1R2 cosα + √ R2R3 cos β+ + √ R3R4 cos γ+ √ R4R1 cos δ) ≤ R1 +R2 +R3 +R4. Chú ý 3.2. Ta luôn có l1 ≥ r1, l2 ≥ r2, l3 ≥ r3, l4 ≥ r4 trong đó r1, r2, r3, r4 lần lượt là khoảng cách từ P tới các cạnh AB,BC,CD,DA của tứ giác lồi ABCD. Do đó, từ bất đẳng thức (3.31) ta có R1 +R2 +R3 +R4 ≥ √ 2 (r1 + r2 + r3 + r4) . (3.32) Bất đẳng thức (3.32) được xem là bất đẳng thức Erdos - Mordell trong tứ giác. www.MATHVN.com - HOANG NGOC QUANG, Yen Bai 87 3.4. Mở rộng bất đẳng thức Erdos-Mordell trong đa giác Bổ đề 3.7. Với mọi dãy số thực x1, x2, · · · , xn ta luôn có cos pi n n∑ k=1 x2k − n−1∑ k=1 xkxk+1 + x1xn = = n−2∑ k=1 1 2 sin kpin sin (k+1)pi n ( sin (k + 1) pi n xk − sin kpi n xk+1 + sin pi n xn )2 . Chứng minh. Bằng cách so sánh các hệ số của x2k và xkxk+1. Chẳng hạn, hệ số của x2k là sin (k+1)pin 2 sin kpin + sin (k−1)pin 2 sin kpin = 2 sin kpin cos pi n 2 sin kpin = cos pi n , với k = 2, 3, · · · , n− 2 và hệ số của x21 là sin 2pin 2 sin pin = cos pin , hệ số của x 2 n−1 là sin (n−2)pin 2 sin (n−1)pin = sin 2pin 2 sin pin = cospin và hệ số của x 2 n là n−2∑ k=1 sin2 pin 2 sin kpin sin (k+1)pi n = n−2∑ k=1 sin pin 2 ( cot kpi n − cot (k + 1) pi n ) = sin pin 2 ( cot pi n − cot (n− 1) pi n ) = cos pi n . Hệ số của xkxk+1 là − sin (k+1)pin sin kpin 2 sin kpin sin (k+1)pi n = −1 với k = 1, 2, · · · , n−2 và hệ số của xn−1xn là −2 sin (n−2)pin sin pin 2 sin (n−2)pin sin (n−1)pi n = − sin pin sin pin = −1. Hệ số của x1xn là 2 sin 2pin sin pi n 2 sin pin sin 2pi n = 1 và hệ số của xkxn với k = 2, 3, · · · , n− 2 là n−2∑ k=2 2 sin (k+1)pin sin pi n 2 sin kpin sin (k+1)pi n − n−3∑ k=1 2 sin kpin sin pi n 2 sin kpin sin (k+1)pi n = n−2∑ k=2 sin pin sin kpin = n−3∑ k=2 sin pin sin (k+1)pin = 0. www.MATHVN.com - HOANG NGOC QUANG, Yen Bai 88 Hệ quả 3.3. Với mọi dãy số thực x1, x2, · · · , xn ta luôn có cos pi n n∑ k=1 x2k − n−1∑ k=1 xkxk+1 + x1xn ≥ 0. (3.33) Chứng minh. Vì sin kpin > 0, sin (k+1)pi n > 0 với mọi k = 1, 2, · · · , n nên n−2∑ k=1 1 2 sin kpin sin (k+1)pi n ( sin (k + 1) pi n xk − sin kpi n xk+1 + sin pi n xn )2 ≥ 0. Từ bất đẳng thức trên và bổ đề 3.7 được bất đẳng thức (3.33) Bổ đề 3.8. (Wolstenholme-Lenhard) Với mọi dãy số thực R1, R2, · · · , Rn,Rn+1 = R1 và mọi dãy θ1, θ2, · · · , θn thỏa mãn n∑ k=1 θk = (2r + 1) pi, r ∈ N, bất đẳng thức sau luôn đúng cos pi n n∑ k=1 R2k ≥ n∑ k=1 RkRk+1 cos θk. (3.34) Chứng minh. Áp dụng hệ quả 3.3, tương tự ta có cos pi n n∑ k=1 y2k − n−1∑ k=1 ykyk+1 + y1yn ≥ 0. (3.35) với mọi dãy số thực y1, y2, · · · , yn. Đặt xk = Rk cos βk và yk = Rk sin βk trong đó Rk ∈ R và βk ∈ R . Cộng theo vế hai bất đẳng thức (3.33) và (3.35) ta được cos pi n n∑ k=1 R2k − n−1∑ k=1 RkRk+1 [cosβkcosβk+1 + sin βk sin βk+1] +R1Rn [cosβ1cosβn + sin β1 sin βn] ≥ 0 hay cos pin n∑ k=1 R2k − n−1∑ k=1 RkRk+1 cos (βk − βk+1) +R1Rncos (β1 − βn) ≥ 0. Đặt βk − βk+1 = θk với k = 1, 2, · · · , n − 1 và β1 − βn = (2r + 1) pi − θn, r ∈ N ta được bất đẳng thức cos pi n n∑ k=1 R2k ≥ n∑ k=1 RkRk+1 cos θk, trong đó Rk ∈ R, θk ∈ R và n∑ k=1 θk = (2r + 1) pi, r ∈ N. www.MATHVN.com - HOANG NGOC QUANG, Yen Bai 89 Định lý 3.7. (Bất đẳng thức Erdos-Mordell cho đa giác) Cho A1A2 · · ·An, (n ≥ 3) là đa giác lồi và P là một điểm tùy ý nằm trong nó. Gọi R1, R2, · · · , Rn lần lượt là khoảng cách tù P tới các đỉnh A1, A2, · · · , An và r1, r2, · · · , rn lần lượt là khoảng cách từ điểm P tới các cạnh A1A2, A2A3, · · · , AnA1. Ta luôn có bất đẳng thức n∑ k=1 Rk ≥ 1 cos pi n n∑ k=1 rk, (3.36) Đẳng thức xảy ra khi và chỉ khi A1A2 · · ·An là đa giác đều và P là tâm của nó. Chứng minh. Gọi l1, l2, · · · , ln lần lượt là độ dài các đường phân giác của các góc Â1PA2, Â2PA3, · · · , ÂnPA1. Đặt Â1PA2 = 2θ1, Â2PA3 = 2θ2, · · · , ÂnPA1 = 2θn, ta có θ1 + θ2 + · · ·+ θn = pi. Theo công thức đường phân giác và bất đẳng thức AM −GM ta có l1 = 2R1R2 R1 +R2 cos θ1 ≤ √ R1R2 cos θ1, l2 = 2R2R3 R2 +R3 cos θ2 ≤ √ R2R3 cos θ2, ........................................................ ln = 2RnR1 Rn +R1 cos θn ≤ √ R4R1 cos θn. Từ n bất đẳng thức trên và áp dụng bổ đề 3.8 ta có 1 cos 1 n n∑ k=1 lk ≤ 1 cos 1 n n∑ k=1 √ RkRk+1. cos θk ≤ n∑ k=1 Rk. Vì l1 ≥ r1, l2 ≥ r2, · · · , ln ≥ rn nên từ bất đẳng thức trên, ta có n∑ k=1 Rk ≥ 1 cos pi n n∑ k=1 rk. Đẳng thức xảy ra khi và chỉ khi A1A2 · · ·An là đa giác đều và P là tâm của nó. www.MATHVN.com - HOANG NGOC QUANG, Yen Bai 90 3.5. Mở rộng bất đẳng thức Erdos-Mordell trong tứ diện Bài toán 3.9. Cho tứ diện ABCD và P là một điểm tùy ý nằm trong nó. Gọi d1, d2, d3, d4 lần lượt là khoảng cách từ P đến các đỉnh A,B,C,D và h1, h2, h3, h4 lần lượt là khoảng cách từ điểm P đến các mặt (BCD), (CDA), (DAB), (ABC). Chứng minh rằng d1 + d2 + d3 + d4 ≥ 2 (√ h1h2 + √ h1h3 + √ h1h4 + √ h2h3 + √ h2h4 + √ h3h4 ) . (3.37) Giải. Gọi Hình 3.12 ha là độ dài đường cao của tứ diện hạ từ đỉnh A xuống mặt đáy BCD. S1, S2, S3, S4 lần lượt là diện tích của các tam giác BCD, CDA,DAB, ABC. V1, V2, V3, V4, V lần lượt là thể tích của các khối tứ diện PBCD, PCDA, PDAB, PABC, ABCD. Ta có d1 + h1 ≥ ha ⇒ d1S1 + h1S1 ≥ haS1 ⇒ d1S1 + 3V1 ≥ 3V = 3 (V1 + V2 + V3 + V4) ⇒ d1S1 ≥ 3V2 + 3V3 + 3V4 = h2S2 + h3S3 + h4S4 ⇒ d1 ≥ h2S2 S1 + h3 S3 S1 + h4 S4 S1 . Tương tự d2 ≥ h1S1 S2 + h3 S3 S2 + h4 S4 S2 , d3 ≥ h1S1 S3 + h2 S3 S3 + h4 S4 S3 , d4 ≥ h1S1 S4 + h2 S3 S4 + h4 S3 S4 . Cộng theo vế 4 bất đẳng thức trên và áp dụng bất đẳng thức AM−GM được d1 + d2+d3 + d4 ≥ ( h1 S1 S2 + h2 S2 S1 ) + ( h1 S1 S3 + h3 S3 S1 ) + ( h1 S1 S4 + h4 S4 S1 ) + ( h2 S2 S3 + h3 S3 S2 ) + ( h2 S2 S4 + h4 S4 S2 ) + ( h3 S3 S4 + h4 S4 S3 ) . ≥2 (√ h1h2 + √ h1h3 + √ h1h4 + √ h2h3 + √ h2h4 + √ h3h4 ) Nhận xét 3.2. Đặc biệt hóa điểm P và tứ diện ABCD ta thu được các kết quả sau: www.MATHVN.com - HOANG NGOC QUANG, Yen Bai 91 + Nếu P ≡ I (I là tâm mặt cầu nội tiếp tứ diện ABCD) thì từ (3.37) ta có d1 + d2 + d3 + d4 ≥ 12r, (3.38) trong đó r là bán kính mặt cầu nội tiếp tứ diện. + Nếu ABCD là tứ diện gần đều (AB = CD,BC = AD,AC = BD) có diện tích một mặt là S thì từ (3.37) ta có d1 + d2 + d3 + d4 ≥ 3h, (3.39) trong đó h là chiều cao của tứ diện gần đều ABCD. + Nếu ABCD là tứ diện đều, cạnh a thì từ (3.39) ta có d1 + d2 + d3 + d4 ≥ a √ 6. (3.40) + Nếu ABCD là tứ diện vuông tại A (tức là AB⊥AC, AB⊥AD, AC⊥AD) và P ≡ I thì d1 = IA = √ 3r kết hợp với (3.38) ta được d2 + d3 + d4 ≥ ( 12− √ 3 ) r. (3.41) www.MATHVN.com - HOANG NGOC QUANG, Yen Bai 92 Chương 4 Các bất đẳng thức có trọng Chương này trình bày một số bất đẳng thức có trọng và làm đặc biệt hóa để thu được các bất đẳng thức hình học mới liên quan đến khoảng cách từ một điểm nằm trong tam giác đến các đỉnh và các cạnh. Nội dung chủ yếu của chương được hình thành từ các tài liệu [9], [13] và [14]. 4.1. Bất đẳng thức dạng Hayashi và các hệ quả 4.1.1. Bất đẳng thức Hayashi Định lý 4.1. (Bất đẳng thức Hayashi) Cho M là một điểm tùy ý trong mặt phẳng chứa tam giác ABC với độ dài các cạnh là a, b, c. Khi đó aMB.MC + bMC.MA+ cMA.MB ≥ abc. (4.1) Đây là một định lý có cách chứng minh hình học khá phức tạp. Sau đây bằng cách dùng số phức sẽ cho ta một cách chứng minh định lý rất ngắn gọn và ấn tượng. Chỉ cần dùng một đồng nhất thức đại số quen thuộc và biểu diễn hình học của số phức. Chứng minh. Ta đặt tam giác ABC lên mặt phẳng phức và gọi m,α, β, γ tương ứng là toạ vị của M,A,B,C. Từ đồng nhất đại số (m− β)(m− γ) (α− β)(α− γ) + (m− γ)(m− α) (β − γ)(β − α) + (m− α)(m− β) (γ − α)(γ − β) = 1, (4.2) đúng với mọi α, β, γ đôi một khác nhau và với mọi m. Đồng nhất thức này có thể chứng minh bằng cách để ý vế trái là một tam thức bậc hai theo m và có giá trị bằng 1 tại ba điểm phân biệt α, β, γ , do đó đồng nhất 1. Từ đồng nhất (4.2) chuyển qua modul ta được |m− β| |m− γ| |α− β| |α− γ| + |m− γ| |m− α| |β − γ| |β − α| + |m− α| |m− β| |γ − α| |γ − β| ≥ 1. (4.3) www.MATHVN.com - HOANG NGOC QUANG, Yen Bai 93 Theo cách đặt trên ta được MA = |m − α|,MB = |m − β|, |MC| = |m− γ|, c = AB = |α− β|, a = BC = |β − γ|, b = CA = |γ − α|. Khi đó (4.3) tương đương với MB.MC bc + MC.MA ca + MA.MB ab ≥ 1.  Chú ý 4.1. Trường hợp xảy ra dấu đẳng thức ta sẽ xét đến trong bài toán 4.1 ở mục dưới. Định lý 4.2. ChoM là một điểm tùy ý trong mặt phẳng chứa tam giác ABC với độ dài các cạnh là a, b, c. Khi đó aMA2 + bMB2 + cMC2 ≥ abc. (4.4) Chứng minh. Xét gốc của mặt phẳng phức ở M và gọi α, β, γ là tọa vị các đỉnh A,B,C của tam giác ABC. Từ đồng nhất thức α2 (α− β) (α− γ) + β2 (β − α) (β − γ) + γ2 (γ − α) (γ − β) = 1, (4.5) đúng với mọi α, β, γ đôi một khác nhau. Chuyển qua modul ta được 1 = ∣∣∣∣∣∑ cyc α2 (α− β) (α− γ) ∣∣∣∣∣ ≤∑ cyc |α|2 |α− β| |α− γ| . (4.6) Theo cách đặt trên ta được |α| = PA, |β| = PB, |γ| = PC và |β − γ| = a, |γ − α| = b, |α− β| = c, bất đẳng thức (4.6) tương đương với bất đẳng thức aMA2 + bMB2 + cMC2 ≥ abc.  Định lý 4.3. ChoM là một điểm tùy ý trong mặt phẳng chứa tam giác ABC với độ dài các cạnh là a, b, c. Khi đó aMA3 + bMB3 + cMC3 ≥ abc.MG. (4.7) Trong đó G là trọng tâm của tam giác ABC. Chứng minh. Từ đồng nhất thức x3 (y − z) + y3 (z − x) + z3 (x− y) = (x− y) (y − z) (z − x) (x+ y + z) , (4.8) đúng với mọi số phức x, y, z. Chuyển qua modul, ta được |x|3 |y − z|+ |y|3 |z − x|+ |z|3 |x− y| ≥ |x− y| |y − z| |z − x| |x+ y + z| . (4.9) Xét gốc của mặt phẳng phức ở G và gọi α, β, γ,m là tọa vị của các điểm A,B,C,M tương ứng. Trong bất đẳng thức (4.9) xét x = m − α, y = m− β, z = m− γ ta thu được bất đẳng thức (4.7).  www.MATHVN.com - HOANG NGOC QUANG, Yen Bai 94 4.1.2. Các hệ quả của bất đẳng thức hyashi Hệ quả 4.1. (Bất đẳng thức Euler) Kí hiệu R, r lần lượt là bán kính đường tròn ngoại tiếp và nội tiếp tam giác ABC. Khi đó R ≥ 2r. Chứng minh. Xét trường hợp M ≡ O (trong đó O là tâm đường tròn ngoại tiếp tam giác ABC), bất đẳng thức (4.1) tương đương với R2 (a+ b+ c) ≥ abc. Do đó R2 ≥ abc a+ b+ c = abc 2p = 4R.S 2.Sr = 2Rr, từ đó R ≥ 2r.  Hệ quả 4.2. Kí hiệuma,mb,mc lần lượt là độ dài các đường trung tuyến xuất phát từ các đỉnh A,B,C của tam giác ABC. Khi đó mamb ab + mbmc bc + mcma ca ≥ 9 4 . Chứng minh. Xét trường hợp M ≡ G (G là trọng tâm của tam giác ABC), từ bất đẳng thức (4.1) ta suy ra bất đẳng thức cần chứng minh. Dấu đẳng thức xảy ra khi và chỉ khi tam giác ABC đều.  4.1.3. Bài toán áp dụng Bài toán 4.1. (Olympic toán Trung Quốc 1998) Cho ABC là tam giác nhọn và cho P là một điểm trong nó. Chứng minh rằng aPB.PC + bPC.PA+ bPA.PB ≥ abc. (4.10) Đẳng thức xảy ra nếu và chỉ nếu P là trực tâm của tam giác ABC. Giải. Giả sử P là gốc của mặt phẳng phức và đặt α, β, γ là tọa vị của A,B,C, tương ứng. Khi đó với mọi α, β, γ ta có đồng nhất thức sau αβ (α− β) + βγ (β − γ) + γα (γ − α) = − (α− β) (β − γ) (γ − α) . (4.11) Lấy modul hai vế đồng nhất trên ta được bất đẳng thức |αβ (α− β)|+ |βγ (β − γ)|+ |γα (γ − α)| ≥ |(α− β) (β − γ) (γ − α)| . (4.12) Theo cách đặt trên ta có PA = |α| , PB = |β| , PC = |γ| , c = |α− β| , a = |β − γ| , b = |γ − α|. Do đó được bất đẳng thức aPB.PC + bPC.PA+ bPA.PB ≥ abc. www.MATHVN.com - HOANG NGOC QUANG, Yen Bai 95 Bây giờ, ta xác định xem khi nào đẳng thức xảy ra. Đặt z1 = αβ (α− γ) (β − γ) , z2 = βγ (β − α) (γ − α) , z3 = γα (γ − β) (α− β) , với cách đặt này, từ (4.11) và (4.12) ta có các đẳng thức và bất đẳng thức sau z1 + z2 + z3 = 1, |z1|+ |z2|+ |z3| ≥ 1. Vậy đẳng thức xảy ra khi và chỉ khi z1, z2, z3 là các số thực dương. Ta chứng minh z1, z2, z3 là các số thực dương khi và chỉ khi P là trực tâm của tam giác ABC. Thật vậy, giả sử z1, z2, z3 là các số thực dương. Vì −z1z2 z3 = ( β γ − α )2 ,−z2z3 z1 = ( γ α− β )2 ,−z3z1 z2 = ( α β − γ )2 . Từ số phức αβ−γ có bình phương là số một số thực âm nên là các số thuần ảo; vậy góc tạo bởi giữa các véc tơ biểu diễn α là −→ PA và véc tơ biểu diễn β − γ là −−→BC là góc có số đo 900, suy ra PA⊥BC. Tương tự βγ−α là số thuần ảo, suy ra PB⊥CA. Vậy P là trực tâm của tam giác ABC. Đảo lại, nếu P là trực tâm của tam giác nhọn ABC. Điểm P ở bên trong tam giác và PA⊥BC nên góc tạo bởi giữa véc tơ −→PA và −−→BC có số đo −900, suy ra tồn tại số thực dương r1 sao cho αβ−γ = −r1i. Tương tự, tồn tại các số thực dương r2, r3 sao cho β γ−α = −r2i, γα−β = −r3i. Vậy z1, z2, z3 là các số thực dương. Từ các kết quả trên ta kết luận đẳng thức xảy ra khi và chỉ khi P là trực tâm của tam giác ABC. Bài toán 4.2. Cho G là trọng tâm của tam giác ABC . Gọi R1, R2, R3, R là bán kính đường tròn ngoại tiếp các tam giác GBC,GCA,GAB,ABC tương ứng. Chứng minh rằng R1 +R2 +R3 ≥ 3R. Giải. Áp dụng định lí 4.1 với G là trọng tâm của tam giác ABC. Ta có aGB.GC + bGC.GA+ cGA.GB ≥ abc, (4.13) trong đó a, b, c là độ dài các cạnh của tam giác ABC. Mặt khác, ta có aGB.GC = 4R1. [GBC] = 4 3R1. [ABC]. Tương tự bGC.GA = 43R2. [ABC] , cGA.GB = 4 3R3. [ABC] . Từ đó (4.13) tương đương với 43 (R1 +R2 +R3) . [ABC] ≥ 4R. [ABC]. Suy ra R1+R2+R3 ≥ 3R. www.MATHVN.com - HOANG NGOC QUANG, Yen Bai 96 Bài toán 4.3. (Chọn đội tuyển IMO Rumani 2004) Cho tam giác ABC và P là một điểm trong nó. Gọi R1, R2, R3, R là bán kính đường tròn ngoại tiếp các tam giác PBC,PCA, PAB,ABC tương ứng. Các đường thẳng PA, PB, PC lần lượt cắt các cạnh BC,CA,AB tại A1, B1, C1. Ký hiệu k1 = PA1 AA1 , k2 = PB1 BB1 , k2 = PC1 CC1 . Chứng minh rằng k1R1 + k2R2 + k3R3 ≥ R. Giải. Hình 4.1 Ta có k1 = PA1 AA1 = [PA1B][AA1B] = [PA1C] [AA1C] = [PA1B]+[PA1C] [AA1B]+[AA1C] = [PBC][ABC] , tương tự k2 = [PCA] [ABC] , k3 = [PAB] [ABC] . Mặt khác [ABC] = abc4R , [PBC] = aPB.PC 4R1 , [PCA] = bPC.PA4R2 , [PAB] = cPA.PB 4R3 . Bất đẳng thức Hayashi áp dụng cho tam giác ABC và điểm P nằm trong nó tương đương với bất đẳng thức sau aPB.PC 4R1 .R1 + bPC.PA 4R2 .R2 + cPA.PB 4R3 .R3 ≥ R.abc 4R . Do đó, ta được [PBC] .R1 + [PCA] .R2 + [PAB] .R3 ≥ R. [ABC] hay [PBC] [ABC] .R1 + [PCA] [ABC] .R2 + [PAB] [ABC] .R3 ≥ R. Từ đó, được bất đẳng thức cần chứng minh k1R1 + k2R2 + k3R3 ≥ R. 4.2. Bất đẳng thức Weizenbock suy rộng và các hệ quả 4.2.1. Bất đẳng thức Weizenbock suy rộng Chúng ta đã biết các bất đẳng thức sau đây: Giả sử a, b, c là độ dài ba cạnh, còn S là diện tích của một tam giác thì a2 + b2 + c2 ≥ (a− b)2 + (b− c)2 + (c− a)2 + 4 √ 3S. (4.14) a2 + b2 + c2 ≥ 4 √ 3S. (4.15) Bất đẳng thức (4.14) được gọi là bất đẳng thức Hadwiger - Finsler, còn bất đẳng thức (4.15) là bất đẳng thức Weizenbock là hệ quả của bất www.MATHVN.com - HOANG NGOC QUANG, Yen Bai 97 đẳng thức Hadwiger - Finsler. Trong mục này xét sự mở rộng của bất đẳng thức Weizenbock. Định lý 4.4. Cho x, y, z là các số thực thỏa mãn các điều kiện x+y, y+ z, z + x, xy + yz + zx ≥ 0. Đặt a, b, c là độ dài ba cạnh và S là diện tích của tam giác ABC. Khi đó xa2 + yb2 + zc2 ≥ 4√xy + yz + zx.S. (4.16) Chứng minh. Áp dụng định lý hàm số cosin c2 = a2 + b2 − 2ab cosC và công thức diện tích S = 12ab sinC, ta có xa2 + yb2 + zc2 ≥ 4√xy + yz + zx.S ⇔xa2 + yb2 + z (a2 + b2 − 2ab cosC) ≥ √xy + yz + zx.2ab sinC ⇔ (x+ z) a2 + (y + z) b2 ≥ 2ab [√xy + yz + zx sinC + z cosC] ⇔ (x+ z) a b + (y + z) b a ≥ 2 [sinC√xy + yz + zx+ z cosC] . (4.17) Áp dụng bất đẳng thức Cauchy - Schwarz, ta có[ sinC √ xy + yz + zx+ z cosC ]2 ≤ (xy + yz + zx+ z2) (sin2C + cos2C) = xy + yz + zx+ z2 = (x+ z) (y + z) . Mặt khác [ (x+ z) a b + (y + z) b a ]2 ≥ 4 (x+ z) (y + z) . Do đó (4.17) đúng. Dấu đẳng thức xảy ra khi và chỉ khi (x+ z) a b = (y + z) b a cosC z = sinC√ xy + yz + zx ⇔  a√ y + z = b√ x+ z cos2C z2 = sin2C xy + yz + zx = sin2C + cos2C xy + yz + zx+ z2 = 1 (x+ z) (y + z) . www.MATHVN.com - HOANG NGOC QUANG, Yen Bai 98 Thay b và cosC tương ứng vào biểu thức c2 = a2 + b2 − 2bc cosC ta được c2 = a2 + a2 x+ z y + z − 2a2 √ x+ z y + z z√ (x+ z) (y + z) ⇔ ( c a )2 = 1 + x+ z y + z − 2 z y + z ⇔ c a = √ x+ y y + z ⇔ a√ y + z = c√ x+ y . Vậy đẳng thức xảy ra khi và chỉ khi a√ y+z = b√ x+ z = c√ x+ y . Bất đẳng thức (4.16) được gọi là bất đẳng thức weizenbock suy rộng. Tiếp theo, ta sẽ mở rộng bất đẳng thức Hadwiger - Finsler. Định lý 4.5. Cho x, y, z là các số thực dương. Bất đẳng thức sau luôn đúng (y + z) a2 + (z + x) b2 + (x+ y) c2 ≥8√xy + yz + zxS + (x+ y) (a− b)2 + (y + z) (b− c)2 + (z + x) (c− a)2 . (4.18) Chứng minh. Hình 4.2 Gọi A′, B′, C ′ lần lượt là tâm đường tròn bàng tiếp các góc A,B,C. Dễ dàng chứng minh được bộ ba điểm (A′B,C ′) , (B′, C, A′) , (C ′, A,B′) thẳng hàng. Gọi K,H là chân đường cao hạ từ C ′ xuống AB và từ B′ xuống AC. Áp dụng định lí 4.4 vào tam giác A′B′C ′ ta có zA′B′2 + xB′C ′2 + yC ′A′2 ≥ 4 √ xy + yz + zxSA′B′C ′. Ta có B′C ′ =B′A+ AC ′ = AH cos ( pi 2 − A2 ) + AK cos ( pi 2 − A2 ) = AH +HK cos ( pi 2 − A2 ) www.MATHVN.com - HOANG NGOC QUANG, Yen Bai 99 = p− c+ p− b cos ( pi 2 − A2 ) = a sin A2 = 2R sinA sin A2 = 4R cos A 2 . Tương tự A′B′ = 4R cos C 2 , C ′A′ = 4R cos B 2 . Do đó SA′B′C ′ = 1 2 .B′C ′.A′B′. sinB′ = 1 2 .4R cos A 2 .4R cos C 2 . cos B 2 =8R2 cos A 2 cos C 2 cos B 2 = R2 sin A sinB sinC sin A2 sin B 2 sin C 2 = 1 8R . 2R sinA.2R sinB.2R sinC sin A2 sin B 2 sin C 2 = abc 2r = abc 4R . 2R r = 2S.R r . Theo công thức vòng tròn bàng tiếp, ta có ra + rb = p [ tan A 2 + tan B 2 ] = p ( r p− a + r p− b ) = prc (p− a) (p− b) . Mặt khác, từ công thức diện tích tam giác lại có pr. abc 4R = p (p− a) (p− b) (p− c)⇒ prc (p− a) (p− b) = 4R p (p− c) ab , nên ra + rb = prc (p− a) (p− b) = 4R p (p− c) ab = 4R cos2 C 2 . Tương tự rb + rc = 4R cos 2 A 2 , rc + ra = 4R cos 2 B 2 . Từ các kết quả trên ta có zA′B′2 + xB′C ′2 + yC ′A′2 =z.16R2 cos2 C 2 + x.16R2 cos2 A 2 + y.16R2 cos2 B 2 =4R ( z.4R. cos2 C 2 ) + 4R ( x.4R. cos2 A 2 ) + 4R ( y.4R. cos2 B 2 ) =4R [z (ra + rb) + x (rb + rc) + y (rc + ra)] . Suy ra 4R [z (ra + rb) + x (rb + rc) + y (rc + ra)] ≥ 4√xy + yz + zx.2SRr . Biết rằng ra(p− a) = rb(p− b) = rc(p− c) = pr = S, nên bất đẳng thức trên tương đương với ⇔r [ra (y + z) + rb (z + x) + rc (z + x)] ≥ 2S √ xy + yz + zx www.MATHVN.com - HOANG NGOC QUANG, Yen Bai 100 ⇔4 (y + z) S 2 p (p− a) + 4 (z + x) S2 p (p− b) + 4 (x+ y) S2 p (p− c) ≥ 8S√xy + yz + zx ⇔4 (y + z) (p− b) (p− c) + 4 (z + x) (p− c) (p− a) + 4 (x+ y) (p− a) (p− b) ≥ 8S√xy + yz + zx ⇔ (y + z) a2 + (z + x) b2 + (x+ y) c2 ≥ 8S√xy + yz + zx+ (x+ y) (a− b)2 + (y + z) (b− c)2 + (z + x) (c− a)2. Nhận xét 4.1. Ở bất đẳng thức (4.18) nếu đặt y + z = x1, z + x = y1, x+ y = z1 thì bất đẳng thức sẽ trở thành x1a 2 + y1b 2 + z1c 2 ≥4 √ x21 + y 2 1 + z 2 1 − (x1 − y1)2 − (y1 − z1)2 − (z1 − x1)2S +x1(b− c)2 + y1(c− a)2 + z1(a− b)2. (4.19) Như vậy bất đẳng thức (4.19) đúng với x1, y1, z1 là độ dài ba cạnh của một tam giác. Vấn đề đặt ra là liệu bất đẳng thức (4.19) có còn đúng với mọi x1, y1, z1 là các số thực dương hay không ? Câu trả lời là có, được thể hiện thông qua định lí sau đây: Định lý 4.6. Với x, y, z là các số thực dương. Chứng minh rằng xa2 + yb2 + zc2 ≥4 √ x2 + y2 + z2 − (x− y)2 − (y − z)2 − (z − x)2S +x(b− c)2 + y(c− a)2 + z(a− b)2. (4.20) Chứng minh. Áp dụng bất đẳng thức (4.16) cho tam giác A′B′C ′ ta có (−x+ y + z)B′C ′2 + (x− y + z)C ′A′2 + (x+ y − z)A′B′2 ≥4 √ x2 + y2 + z2 − (x− y)2 − (y − z)2 − (z − x)2.SA′B′C ′. Mặt khác, ta lại có (−x+ y + z)B′C ′2 + (x− y + z)C ′A′2 + (x+ y − z)A′B′2 =(−x+ y + z) 16R2 cos2 A 2 + (x− y + z) 16R2 cos2 B 2 + (x+ y − z) 16R2 cos2 C 2 =4R [(−x+ y + z) (rb + rc) + (x− y + z) (rc + ra) + (x+ y − z) (ra + rb)] =8R (xra + yrb + zrc) = 8R r (xrra + yrrb + zrrc) www.MATHVN.com - HOANG NGOC QUANG, Yen Bai 101 = 8R r [x (p− b) (p− c) + y (p− c) (p− a) + z (p− a) (p− b)] . và 4 √ x2 + y2 + z2 − (x− y)2 + (y − z)2 + (z − x)2.SA′B′C ′ =4 √ x2 + y2 + z2 − (x− y)2 + (y − z)2 + (z − x)2.2S.R r . Suy ra xa2 + yb2 + zc2 ≥ 4 √ x2 + y2 + z2 − (x− y)2 − (y − z)2 − (z − x)2S +x(b− c)2 + y(c− a)2 + z(a− b)2.  4.2.2. Các hệ quả của bất đẳng thức Weizenbock suy rộng Hệ quả 4.3. Với mọi số thực dương x, y, z. Khi đó bất đẳng thức sau luôn đúng xab+ ybc+ zca ≥ 4√xy + yz + zx.S (4.21) Chứng minh. Vì bất đẳng thức Weizenbock suy rộng (4.16) đúng với mọi số thực dương x, y, z nên thay x→ x ba , y → y cb , z → z ac ta được xab+ ybc+ zca ≥ 4 √ xy c a + yz a b + zx b c .S hay (xab+ ybc+ zca)2 ≥ 16S2 ( xy c a + yz a b + zx b c ) . (4.22) Lại thay x → z c a , y → xa b , z → yb c vào bất đẳng thức Weizenbock suy rộng, sau đó bình phương hai vế , ta được bất đẳng thức (xab+ ybc+ zca)2 ≥ 16S2 ( xy a c + yz b a + zx c b ) . (4.23) www.MATHVN.com - HOANG NGOC QUANG, Yen Bai 102 Cộng theo vế 2 bất đẳng thức (4.22) và (4.23), rồi áp dụng bất đẳng thức AM −GM , ta được 2 (xab+ ybc+ zca)2 ≥ 16S2 [ xy (a c + c a ) + yz ( b a + a b ) + zx ( b c + c b )] ≥ 2.16S2 (xy + yz + zx) . Suy ra xab+ ybc+ zca ≥ 4√xy + yz + zx.S.  Trong các bất đẳng thức (4.16) và (4.21) có xuất hiện những biến số x, y, z dương tùy ý. Ta nghĩ đến việc thay các số x, y, z bằng các biến MA,MB,MC sẽ thu được một số kết quả sau Hệ quả 4.4. Cho M là một điểm tùy ý trong tam giác ABC. Khi đó bất đẳng thức sau luôn đúng aMA+ bMB + cMC ≥ 4S. (4.24) Chứng minh. Áp dụng bất đẳng thức Weizenbock suy rộng với x = MA a , y = MB b , z = MC c , ta có aMA + bMB + cMC = MA a a 2 + MBb b 2 + MC c c 2 ≥ 4 √ MA a MB b + MB b MC c + MC c MA a .S. Mặt khác, ta có MA a MB b + MB b MC c + MC c MA a ≥ 1 nên aMA+ bMB + cMC ≥ 4S.  Hệ quả 4.5. Cho M là một điểm tùy ý trong tam giác ABC. Khi đó bất đẳng thức sau luôn đúng bc a MA+ ca b MB + ab c MC ≥ 4S. (4.25) Chứng minh. Áp dụng bất đẳng thức (4.21) hệ quả 4.3 với x = MC c , y = MA a , z = MB b , ta có bc aMA + ca bMB + ab cMC ≥ 4 √ MA a MB b + MB b MC c + MC c MA a .S ≥ 4S. Hệ quả 4.6. Cho tam giác ABC với độ dài các cạnh a, b, c diện tích S và tam giác A′B′C ′ với độ dài các cạnh a′, b′, c′ diện tích S ′ ; M là một điểm tùy ý trong mặt phẳng. Khi đó bất đẳng thức sau luôn đúng a′2 a MA+ b′2 b MB + c′2 c MC ≥ 4S ′. (4.26) www.MATHVN.com - HOANG NGOC QUANG, Yen Bai 103 Chứng minh. Áp dụng bất đẳng thức Weizenbock suy rộng cho tam giác A′B′C ′ với x = MAa , y = MB b , z = MC c , ta có a′2 a MA+ b′2 b MB + c′2 c MC ≥ 4 √ MA a MB b + MB b MC c + MC c MA a .S′ ≥ 4S′. Các bất đẳng thức trên là khá khó và đẹp. Thế nhưng ta để ý rằng các hệ số tổng quát x, y, z không xuất hiện. Các hệ số đã được thế bởi các biến MA,MB,MC. Để vẫn xuất hiện các hệ số x, y, z, ta sử dụng bất đẳng thức Hyashi MA.MBab + MB.MC bc + MC.MA ca ≥ 1 thu được kết quả sau Hệ quả 4.7. Cho tam giác ABC với độ dài các cạnh a, b, c và tam giác A′B′C ′ với độ dài các cạnh a′, b′, c′ diện tích S ′. Với M là một điểm tùy ý trong mặt phẳng và x, y, z là các số thực dương, ta có x a′2 a2 MA2 + y b′2 b2 MB2 + z c′2 c2 MC2 ≥ 4 √ xyz x+ y + z .S ′. Chứng minh. Áp dụng bất đẳng thức Weizenbock suy rộng cho tam giác A′B′C ′, ta có x a′2 a2 MA2 + y b′2 b2 MB2 + z c′2 c2 MC2 = x a2 MA2.a′2 + y b2 MB2.b′2 + z c2 MC2.c′2 ≥4 √ yz MB2 b2 . MC2 c2 + zx MC2 c2 . MA2 a2 + xy MA2 a2 . MB2 b2 .S ′. (4.27) Mặt khác áp dụng bất đẳng thức Cauchy - Schwarz, ta có( yz MB2 b2 . MC2 c2 + zx MC2 c2 . MA2 a2 + xy MA2 a2 . MB2 b2 )( 1 yz + 1 zx + 1 xy ) ≥ ( MB.MC bc + MC.MA ca + MA.MB ab )2 ≥ 1. Suy ra yz MB2 b2 . MC2 c2 + zx MC2 c2 . MA2 a2 + xy MA2 a2 . MB2 b2 ≥ xyz x+ y + z . (4.28) Từ (4.27) và (4.28) ta được bất đẳng thức cần chứng minh.  www.MATHVN.com - HOANG NGOC QUANG, Yen Bai 104 Hệ quả 4.8. Cho tam giác ABC, M là một điểm tùy ý trong mặt phẳng. Khi đó bất đẳng thức sau luôn đúng (MA.MB +MB.MC +MC.MA) ( a MA + b MB + c MC ) ≥ 12S. Chứng minh. Áp dụng bất đẳng thức (4.21), ta có MA.MB +MB.MC +MC.MA = MA a . MB b .ab+ MB b . MC c .bc+ MC c . MA a .ca ≥4 √ MA.MB.MC abc ( MA a + MB b + MC c ) .S. Để chứng minh bất đẳng thức đã cho ta cần chứng minh MA.MB.MC abc ( MA a + MB b + MC c )( a MA + b MB + c MC )2 ≥ 9. Thật vậy, ta có( MA a + MB b + MC c )( a MA + b MB + c MC ) ≥ 9 (4.29) và MA.MB.MC abc ( a MA + b MB + c MC ) = MB.MC bc + MC.MA ca + MA.MB ab ≥ 1. (4.30) Từ (4.29) và (4.30) ta có điều phải chứng minh. Hình 4.3 Ta thử mở rộng theo hướng khác để xuất hiện các hệ số x, y, z. Với chú ý rằng: Nếu gọi A′, B′, C ′ là hình chiếu của M trên BC,CA,AB thì MA = 2R.B′C ′ a , MB = 2R.C ′A′ b , MC = 2R.A′B′ c . www.MATHVN.com - HOANG NGOC QUANG, Yen Bai 105 Như thế, ta đã quy các biến MA,MB,MC về độ dài 3 cạnh của một tam giác. Ta có kết quả sau Hệ quả 4.9. Cho tam giác ABC với độ dài các cạnh a, b, c nội tiếp đường tròn (O,R). M là một điểm bất kì trong tam giác. Khi đó với mọi số thực dương x, y, z ta có x MB.MC a + y MC.MA b + z MA.MB c ≥ √xy + yz + zx. ( R2 −OM 2) R . Chứng minh. Gọi A′, B′, C ′ là hình chiếu của M trên BC,CA,AB theo định lý Euler ta có [A′B′C ′] = 1 4 ( 1− OM 2 R2 ) . [ABC] Áp dụng hệ quả 4.3 cho tam giác A′B′C ′ với lưu ý MA = 2R.B′C ′ a , MB = 2R.C ′A′ b , MC = 2R.A′B′ c , ta có √ xy + yz + zx. ( R2 −OM 2) R = 4R. √ xy + yz + zx. [A′B′C ′] [ABC] ≤R.xA ′B′.A′C ′ + yB′C ′.B′A′ + zC ′A′.C ′B′ [ABC] = xMC.MB.cb+ yMA.MC.ac+ zMB.MA.ba 4R [ABC] = xMC.MB.cb+ yMA.MC.ac+ zMB.MA.ba abc =x MB.MC a + y MC.MA b + z MA.MB c . 4.3. Bất đẳng thức Klamkin và các hệ quả 4.3.1. Bất đẳng thức Klamkin Vào năm 1975, M.S. Klamkin đã thiết lập định lý sau đây: Định lý 4.7. Cho ABC là một tam giác tùy ý với độ dài các cạnh lần lượt là a, b, c và P là điểm bất kỳ trong mặt phẳng chứa tam giác. Với các số thực x, y, z ta có (x+ y + z) ( xPA2 + yPB2 + zPC2 ) ≥ yza2 + zxb2 + xyc2. (4.31) www.MATHVN.com - HOANG NGOC QUANG, Yen Bai 106 Chứng minh. Ta có x −→ PA+ y −−→ PB + z −→ PC ≥ 0⇔ (x2PA2 + y2PB2 + z2PC2)+ + ( 2xy −→ PA. −−→ PB + 2yz −−→ PB. −→ PC + 2zx −→ PC. −→ PA ) ≥ 0. (4.32) Theo định lí hàm số cosin ta có 2 −→ PA. −−→ PB = 2PA.PB cos (−→ PA, −−→ PB ) = PA2 + PB2 − c2, 2 −−→ PB. −→ PC = 2PB.PC cos (−−→ PB, −→ PC ) = PB2 + PC2 − a2, 2 −→ PC. −→ PA = 2PC.PA cos (−→ PC, −→ PA ) = PC2 + PA2 − b2. thay các bất đẳng thức trên vào (4.32) ta được thu được bất đẳng thức (x+ y + z) ( xPA2 + yPB2 + zPC2 ) ≥ yza2 + zxb2 + xyc2. Đẳng thức trong (4.31) xảy ra khi và chỉ khi x −→ PA+y −−→ PB+z −→ PC = −→ 0 , tức là P là tâm tỉ cự của hệ điểm {A,B,C}.  4.3.2. Các hệ quả của bất đẳng thức Klamkin Hệ quả 4.10. Trong tam giác ABC với G là trọng tâm, ta có các bất đẳng thức sau 1. PA2 + PB2 + PC2 ≥ a 2 + b2 + c2 3 . (4.33) 2. PA2 + PB2 + PC2 ≥ 4 9 ( m2a +m 2 b +m 2 c ) . (4.34) 3. PA2 + PB2 + PC2 ≥ GA2 +GB2 +GC2. (4.35) Chứng minh. Khi x = y = z, bất đẳng thức 4.31 trở thành 3 ( PA2 + PB2 + PC2 ) ≥ a2 + b2 + c2. Suy ra bất đẳng thức (4.33). Tiếp theo ta biến đổi bất đẳng thức (4.33) như sau PA2 + PB2 + PC2 ≥ 2 ( b2 + c2 )− a2 9 + 2 ( c2 + a2 )− b2 9 + 2 ( a2 + b2 )− c2 9 . Từ đây suy ra các bất đẳng thức (4.34) và (4.35).  www.MATHVN.com - HOANG NGOC QUANG, Yen Bai 107 Hệ quả 4.11. Trong tam giác ABC, ta có các bất đẳng thức sau 1. PA2 a2 + PB2 b2 + PC2 c2 ≥ a 4 + b4 + c4 a2b2 + b2c2 + c2a2 ≥ 1. (4.36) 2. PA2 b2 + PB2 c2 + PC2 a2 ≥ 1. (4.37) 3. PA2 c2 + PB2 a2 + PC2 b2 ≥ 1. (4.38) Chứng minh. Áp dụng bất đẳng thức Klamkin với x = 1 a2 , y = 1 b2 , z = 1 c2 ta thu được bất đẳng thức (4.36). Nếu cho x = 1 b2 , y = 1 c2 , z = 1 a2 thì ta có bất đẳng thức (4.37), còn nếu cho x = 1 c2 , y = 1 a2 , z = 1 b2 ta sẽ có bất đẳng thức (4.38).  Hệ quả 4.12. Trong tam giác ABC, ta có các bất đẳng thức sau 1. m2a a2 + m2b b2 + m2c c2 ≥ 9 4 . (4.39) 2. mamb ab + mbmc bc + mcma ca ≥ 9 4 . (4.40) Chứng minh. Áp dụng bất đẳng thức Klamkin với P ≡ G, x = 1 a2 , y = 1 b2 , z = 1 c2 ta thu được bất đẳng thức (4.39). Nếu cho P ≡ G, x = a ma , y = b mb , z = c mc , ta sẽ có bất đẳng thức (4.40).  Hệ quả 4.13. Trong tam giác ABC, ta có các bất đẳng thức sau 1. cos2A sinB sinC + cos2B sinC sinA + cos2C sinA sinB ≥ 1. (4.41) 2. 4R2 ≥ a 3 + b3 + c3 + abc a+ b+ c . (4.42) 3. 2R− r r ≥ a 3 + b3 + c3 abc . (4.43) Chứng minh. Áp dụng bất đẳng thức Klamkin với x = a, y = b, z = c, P ≡ H (H là trực tâm của tam giác). Để ý rằng HA = |2R cosA| , www.MATHVN.com - HOANG NGOC QUANG, Yen Bai 108 HB = |2R cosB| , HC = |2R cosC| và sử dụng định lý hàm số sin ta dễ dàng nhận được bất đẳng thức (4.41). Sử dụng các biến đổi HA2 = 4R2 cos2A = 4R2 ( 1− sin2A) = 4R2 − a2, HB2 = 4R2 − b2, HC2 = 4R2 − c2, sau một số biến đổi ta nhận được bất đẳng thức (4.42). Sử dụng công thức abc = 4Rrp ta biến đổi bất đẳng thức (4.42) về bất đẳng thức (4.43).  4.4. Bất đẳng thức Jian Liu và các hệ quả 4.4.1. Bất đẳng thức Jian Liu Jian Liu đã chứng minh định lí 4.8 dưới đây. Để chứng minh định lí 4.8 ta cần bổ đề sau: Bổ đề 4.1. Cho ABC là một tam giác tùy ý và P là một điểm bất kỳ trong mặt phẳng chứa tam giác ABC. Nếu bất đẳng thức sau đây f (a, b, c, R1, R2, R3) ≥ 0 (4.44) đúng, thì bất đẳng thức sau cũng đúng f (aR1, bR2, cR3, R2R3, R3R1, R1R2) ≥ 0, (4.45) trong đó R1, R2, R3 lần lượt là khoảng cách từ P đến các đỉnh A,B,C. Chứng minh. Xét phép nghịch đảo N tâm P , hệ số R1.R2.R3, ta có N : A→ A′, B → B′, C → C ′ Khi đó ta có PA′ = PB.PC = R2R3, tương tự PB′ = R3R1, PC ′ = R1R2 và B′C ′ = R1R2R3. BCPB.PC = aR1, tương tự C ′A′ = bR2, A ′B′ = cR3. Vì f (a, b, c, R1, R2, R3) ≥ 0 đúng với mọi tam giác ABC và mọi P nên áp dụng điều kiện này cho tam giác A′B′C ′ và điểm P ta được f (B′C ′, C ′A′, A′B′, PA′, PB′, PC ′) ≥ 0 hay f (aR1, bR2, cR3, R2R3, R3R1, R1R2) ≥ 0.  Định lý 4.8. Cho x, y, z là các số dương. Khi đó, với tam giác ABC tùy ý và P là điểm bất kỳ trong mặt phẳng chứa ∆ABC, bất đẳng thức www.MATHVN.com - HOANG NGOC QUANG, Yen Bai 109 sau đúng R21 x + R22 y + R23 z ≥ aR1 + bR2 + cR3√ xy + yz + zx , (4.46) trong đó R1 = PA,R2 = PB,R3 = PC. Đẳng thức xảy ra khi và chỉ khi ABC là tam giác nhọn, P trùng với trực tâm H và x : y : z = cotA : cotB : cotC. Chứng minh. Nếu P trùng với một trong các đỉnh của ∆ABC, chẳng hạn, P ≡ A, thì PA = 0, PB = c, PC = b và bất đẳng thức (4.46) trở thành tầm thường. Trong trường hợp này, dấu đẳng thức trong (4.46) rõ ràng là không xảy ra. Tiếp theo, giả sử P không trùng với một trong các đỉnh của ∆ABC. Nếu x, y, z là các số dương thì bất đẳng thức (4.31) tương đương với bất đẳng thức sau( xR21 + yR 2 2 + zR 2 3 )( 1 yz + 1 zx + 1 xy ) ≥ a 2 x + b2 y + c2 z . Mặt khác, theo bất đẳng thức Cauchy - Schwarz ta có a2 x + b2 y + c2 z ≥ (a+ b+ c) 2 x+ y + z . Đẳng thức xảy ra khi và chỉ khi x : y : z = a : b : c. Kết hợp hai bất đẳng thức trên ta có bất đẳng thức( xR21 + yR 2 2 + zR 2 3 )( 1 yz + 1 zx + 1 xy ) ≥ (a+ b+ c) 2 x+ y + z . (4.47) Dấu đẳng thức trong (4.47) xảy ra khi và chỉ khi x : y : z = a : b : c và P là tâm của đường tròn nội tiếp tam giác ABC. Bây giờ, áp dụng phép nghịch đảo trong bổ đề 4.1 cho bất đẳng thức (4.47), ta thu được[ x(R2R3) 2 + y(R3R1) 2 + z(R1R2) 2]( 1 yz + 1 zx + 1 xy ) ≥ (aR1 + bR2 + cR3) 2 x+ y + z , hay (R2R3) 2 yz + (R3R1) 2 zx + (R1R2) 2 xy ≥ ( aR1 + bR2 + cR3 x+ y + z )2 . (4.48) www.MATHVN.com - HOANG NGOC QUANG, Yen Bai 110 Thay x→ xR21, y → yR22, z → zR23, ta được 1 yz + 1 zx + 1 xy ≥ ( aR1 + bR2 + cR3 xR21 + yR 2 2 + zR 2 3 )2 . (4.49) Lại một lần nữa thay x → 1 x , y → 1 y , z → 1 z , ta được bất đẳng thức (4.46) cần chứng minh. Nếu đẳng thức trong (4.44) chỉ xảy ra khi P là tâm của đường tròn nội tiếp ∆ABC, thì đẳng thức trong (4.45) xảy ra chỉ khi ∆ABC là nhọn và P là trực tâm của nó. Theo điều này và điều kiện để đẳng thức trong (4.47) xảy ra, ta có đẳng thức trong (4.46) xảy ra khi và chỉ khi ∆ABC là nhọn, P là trực tâm của nó và R1 xa = R2 yb = R3 zc . (4.50) Khi P là trực tâm của tam giác nhọn ABC, ta có R1 : R2 : R3 = cosA : cosB : cosC. Do đó, trong trường hợp này, từ (4.50) ta có x : y : z = cotA : cotB : cotC. Vì vậy, đẳng thức trong (4.46) xảy ra khi và chỉ khi ∆ABC là nhọn, P trùng với trực tâm của nó và xcotA = y cotB = z cotC .  4.4.2. Các hệ quả của bất đẳng thức Jian Liu Hệ quả 4.14. Với P tùy ý nằm trong mặt phẳng chứa tam giác ABC và với mọi số dương x, y, z, bất đẳng thức sau luôn đúng R21 x + R22 y + R23 z ≥ 4S√ xy + yz + zx . (4.51) Đẳng thức xảy ra khi và chỉ khi x : y : z = cotA : cotB : cotC và P là trực tâm của tam giác nhọn ABC. Chứng minh. Từ bất đẳng thức (4.24) và bất đẳng thức Jian Liu (4.46) ta thu được bất đẳng thức cần chứng minh (4.51).  Hệ quả 4.15. Cho tam giác ABC và P là một điểm tùy ý trong mặt phẳng chứa tam giác. Khi đó bất đẳng thức sau đúng R21 +R 2 2 +R 2 3 ≥ 3√ 3 S. (4.52) www.MATHVN.com - HOANG NGOC QUANG, Yen Bai 111 Chứng minh. Trong bất đẳng thức (4.51) cho x = y = z sẽ thu được bất đẳng thức (4.52).  Hệ quả 4.16. Với P tùy ý nằm trong mặt phẳng chứa tam giác ABC và với mọi số dương x, y, z, bất đẳng thức sau luôn đúng aR21 + bR 2 2 + cR 2 3 aR1 + bR2 + cR3 ≥ √ 2Rr. (4.53) Đẳng thức xảy ra khi và chỉ khi tam giác ABC đều và P là trực tâm của nó. Chứng minh. Trong bất đẳng thức Jian Liu (4.46) ta thay x = 1 a , y = 1 b , z = 1 c và sử dụng kết quả 1 bc + 1 ca + 1 ab = 1 2Rr , ta thu được bất đẳng thức cần chứng minh (4.53).  Hệ quả 4.17. (Bất đẳng thức Hayashi) Nếu P là một điểm tùy ý và không trùng với các đỉnh của tam giác ABC, thì R2R3 bc + R3R1 ca + R1R2 ab ≥ 1. (4.54) Đẳng thức xảy ra khi và chỉ khi tam giác ABC nhọn và P là trực tâm của nó. Chứng minh. Trong bất đẳng thức (4.48), thay x = aR1, y = bR2, z = cR3 ta được bất đẳng thức (4.54).  Hệ quả 4.18. Nếu P là một điểm tùy ý và không trùng với các đỉnh của tam giác ABC, thì (R2R3 +R3R1 +R1R2) 2 ( 1 R2R3 + 1 R3R1 + 1 R1R2 ) ≥ 4p2. (4.55) Trong đó p là nửa chu vi của tam giác ABC. Đẳng thức xảy ra khi và chỉ khi tam giác ABC nhọn và P là trực tâm của nó. Chứng minh. Trong (4.48) thay x → xaR1, y → ybR2, z → zcR3 . Khi đó ta được R2R3 yzbc + R3R1 zxca + R1R2 xyab ≥ ( aR1 + bR2 + cR3 xaR1 + ybR2 + zcR3 )2 . (4.56) www.MATHVN.com - HOANG NGOC QUANG, Yen Bai 112 Tiếp tục thay x = 1 a , y = 1 b , z = 1 c , ta được (R2R3 +R3R1 +R1R2) (R1 +R2 +R3) 2 ≥ (aR1 + bR2 + cR3)2. (4.57) Áp dụng phương pháp biến đổi nghịch đảo của bổ đề 4.1 cho bất đẳng thức (4.57), sau đó chia cả hai vế cho (R1R2R3) 2 ta được bất đẳng thức (4.55).  Hệ quả 4.19. Cho P là một điểm nằm trong tam giác ABC, khi đó bất đẳng thức sau luôn đúng (R2R3) 2 r2r3 + (R3R1) 2 r3r1 + (R1R2) 2 r1r2 ≥ 16 9 p2. (4.58) Trong đó p là nửa chu vi của tam giác ABC. Chứng minh. Với P là một điểm tùy ý nằm trong tam giác ABC, ta có các bất đẳng thức sau aR1 ≥ br3 + cr2, bR2 ≥ cr1 + ar3, cR3 ≥ ar2 + br1. Cộng theo vế 3 bất đẳng thức trên với lưu ý là a+ b+ c = 2p, ar1 + br2 + cr3 = 2rp, ta thu được bất đẳng thức aR1 + bR2 + cR3 ≥ 2p (r1 + r2 + r3)− 2rp. Nhân cả hai vế với bất đẳng thức trên với 2, sau đó cộng với bất đẳng thức aR1 + bR2 + cR3 ≥ 4S, với lưu ý là S = rp ta thu được 3 (aR1 + bR2 + cR3) ≥ 4p (r1 + r2 + r3) , (4.59) hay aR1 + bR2 + cR3 r1 + r2 + r3 ≥ 4 3 p. (4.60) Mặt khác, trong bất đẳng thức (4.48) cho x = r1, y = r2, z = r3 ta thu được bất đẳng thức (R1R2) 2 r2r3 + (R3R1) 2 r3r1 + (R1R2) 2 r1r2 ≥ ( aR1 + bR2 + cR3 r1 + r2 + r3 )2 . (4.61) Từ hai bất đẳng thức (4.60) và (4.61) ta được bất đẳng thức (4.58).  www.MATHVN.com - HOANG NGOC QUANG, Yen Bai 113 Hệ quả 4.20. Cho P là một điểm nằm trong tam giác ABC, khi đó bất đẳng thức sau luôn đúng (R1 +R2 +R3) 2 r1 + r2 + r3 ≥ 4√ 3 p. (4.62) Chứng minh. Từ bất đẳng thức (4.57) và (4.60) ta suy ra (R2R3 +R3R1 +R1R2) (R1 +R2 +R3) 2 ≥ 16 9 p2 (r1 + r2 + r3) 2 . Mặt khác, ta có 3 (R2R3 +R3R1 +R1R2) ≤ (R1 +R2 +R3)2 . Từ hai bất đẳng thức trên ta được bất đẳng thức (4.62).  Hệ quả 4.21. Cho P là một điểm nằm trong tam giác ABC, khi đó bất đẳng thức sau luôn đúng R21 ra + R22 rb + R23 rc ≥ 4 3 (r1 + r2 + r3) . (4.63) Chứng minh. Trong bất đẳng thức (4.46) thay x → ra, y → rb, z → rc với lưu ý là rbrc + rcra + rarb = p 2, ta có R21 ra + R22 rb + R23 rc ≥ 1 p (aR1 + bR2 + cR3) . (4.64) Từ bất đẳng thức (4.60) và (4.64) ta được bất đẳng thức (4.63).  Hệ quả 4.22. Cho P là một điểm nằm trong tam giác ABC, khi đó bất đẳng thức sau luôn đúng R21 ra + R22 rb + R23 rc ≥ r1 + r2 + r3 + r. (4.65) Chứng minh. Cộng theo vế các bất đẳng thức (4.59) và aR1+bR2+cR3 ≥ 4S, sau đó chia cả hai vế cho 4, ta có aR1 + bR2 + cR3 ≥ p (r1 + r2 + r3 + r) . (4.66) Từ bất đẳng thức (4.64) và (4.66) ta được bất đẳng thức (4.65).  www.MATHVN.com - HOANG NGOC QUANG, Yen Bai 114 Hệ quả 4.23. Cho P là một điểm nằm trong tam giác ABC. Gọi ha, hb, hc lần lượt là độ dài đường cao hạ từ A,B,C. Khi đó R21 ha + R22 hb + R23 hc ≥ 4r. (4.67) Trong đó r là bán kính đường tròn nội tiếp tam giác ABC. Chứng minh. Ta biết rằng 1ha + 1 hb + 1hc = 1 r hay hahb + hbhc + hcha = hahbhc r = 8S3 abcr = 2S2 rR . Vì R ≥ 2r nên hahb + hbhc + hcha ≤ S 2 r2 . (4.68) Trong (4.51) cho x = ha, y = hb, z = hc và sử dụng (4.68) ta được R21 ha + R22 hb + R23 hc ≥ 4S√ S2 r2 = 4r.  Hệ quả 4.24. Cho P là một điểm nằm trong tam giác ABC. Khi đó bất đẳng thức sau luôn đúng aR21 + bR 2 2 + cR 2 3 ≥ 2 √ S3 4 √ 27 (4.69) Chứng minh. Theo bài toán 1.6 ta có 4 √ 3 ≤ 9abc a+ b+ c hay 1 ab + 1 bc + 1 ca ≤ 3 √ 3 4S . (4.70) Bây giờ trong (4.51) cho x = 1 a , y = 1 b , z = 1 c và sử dụng (4.4.2.) ta được aR21 + bR 2 2 + cR 2 3 ≥ 4S√ 3√3 4S = 2 √ S3 4 √ 27 .  Hệ quả 4.25. Cho P là một điểm nằm trong tam giác ABC. Gọi ma,mb,mc lần lượt là độ dài trung tuyến xuất phát từ A,B,C. Khi đó bất đẳng thức sau luôn đúng maR 2 1 +mbR 2 2 +mcR 2 3 ≥ 4S √ S 4 √ 3 . (4.71) www.MATHVN.com - HOANG NGOC QUANG, Yen Bai 115 Chứng minh. Gọi A1, B1, C1 lần lượt là trung điểm của BC,CA,AB. Dựng tam giác AA1M sao cho M đối xứng với C1 qua B1. Dễ thấy ma,mb,mc là độ dài các cạnh của tam giác MAA1 (ma = AA1,mb = MA1,mc = MA) và [MAA1] = 3 4S. Áp dụng bất đẳng thức ta được 1 mamb + 1 mbmc + 1 mcma ≤ 4S √ S 4 √ 3 = √ 3 S . (4.72) Bây giờ trong (4.51) cho x = 1 ma , y = 1 mb , z = 1 mc và sử dụng bất đẳng thức (4.72) ta được maR 2 1 +mbR 2 2 +mcR 2 3 ≥ 4S√√ 3 S = 4S √ S 4 √ 3 .  www.MATHVN.com - HOANG NGOC QUANG, Yen Bai 116 Kết luận Luận văn đã trình bày và đạt được một số kết quả sau: 1. Trình bày một số bất đẳng thức trong tam giác và trong tứ giác từ cơ bản đến nâng cao. 2. Giới thiệu hai bất đẳng thức quen thuộc và có nhiều áp dụng trong các đề thi học sinh giỏi đó là bất đẳng thức Erdos-modell và bất đẳng thức Ptolemy. Đồng thời cũng xây dựng một số mở rộng của hai bất đẳng thức này. 3. Trình bày một số bất đẳng thức có trọng, các mở rộng và áp dụng, đồng thời đặc biệt hóa để có các bài toán bất đẳng thức hình học mới liên quan đến khoảng cách từ một điểm đến các đỉnh và các cạnh của tam giác. 4. Tác giả đã nêu và chứng minh được một số bổ đề giúp cho việc chứng minh các bất đẳng quan trọng trong luận văn như bất đẳng thức hình bình hành, bất đẳng thức Erdos-Modell mở rộng đối với 2 điểm trong tam giác, mở rộng trong tứ giác và đa giác, bất đẳng thức Jian Liu v.v.. Ngoài ra, khai thác một số bất đẳng thức tác giả đã đặc biệt hóa để thu được nhiều bất đẳng thức hay và đẹp. 5. Luận văn đã chọn lọc giới thiệu một số đề thi học sinh giỏi trong nước, khu vực và quốc tế liên quan đến các bất đẳng thức trong tam giác và trong tứ giác. Các bất đẳng thức hình học áp dụng trực tiếp hai bất đẳng thức nổi tiếng đó là bất đẳng thức Erdos-Modell và bất đẳng thức Ptolemy. www.MATHVN.com - HOANG NGOC QUANG, Yen Bai 117 Tài liệu tham khảo [1] Bộ giáo dục và đào tạo-Hội toán học Việt Nam (2009), Các bài toán chọn lọc - 45 năm Tạp chí Toán học và Tuổi trẻ, NXB Giáo Dục. [2] Nguyễn Minh Hà, Suy nghĩ mới từ một bài toán quen thuộc, Tuyển tập 5 năm tạp chí toán học và tuổi trẻ, NXB Giáo Dục (1995). [3] Vũ Đình Hòa (2005), Bất đẳng thức hình học, NXB Giáo Dục. [4] Phan Huy Khải, Nguyễn Đạo Phương (1994), Hệ thức lượng trong tam giác và tứ giác, NXB Giáo Dục. [5] Nguyễn Văn Nho (2011), Những định lí chọn lọc trong hình học phẳng qua các kì thi Olympic, NXB Đại Học Sư Phạm. [6] Nguyễn Đức Tấn (2000), Chuyên đề bất đẳng thức và cực trị trong hình học phẳng, NXB Giáo dục. [7] Nguyễn Thượng Võ (1989), 200 bài toán chọn lọc về hệ thức lượng trong tam giác, NXB Giáo dục. [8] Trần Nam Dũng, Ptolemy’s inequality and its applications, Kỷ yếu hội nghị khoa học, Việt Trì (2011). [9] Titu Andreescu and Dorin Andrica, Proving some geometric in- equalities by using complex numbers, Educatia Matematica Vol.1, N2, (2005), 19-26. [10] Dusan Djukic, Vladimir Jankovic, Ivan Matic, Nikola Petrovic, The IMO Compendium A Collection of Problems Suggested for the Inter- national Mathematical Olympiads:1959–2004, Springer Publishers, 2004. [11] Yu-Dong Wu, Chun-Lei Yu Zhi-Hua Zhang, A geometric inequality of the generalized Erdos-Modell type, Journal of inequalities in pure and applied mathematics, Vol.10, Iss.4, Ar.106, 2009. [12] Radmila Bulajich Manfrino, José Antonio Gómez Ortega, Rogelio Valdez Delgado, Inequalities A Mathematical Olympiad Approach, Birkhauser Publishers, 2009. www.MATHVN.com - HOANG NGOC QUANG, Yen Bai 118 [13] D.S Mitrinovic, J.E.Pecaric and V.Volenec, Recent Advances in Geometric Inequalities, Kluwer Academic publishers, Dordrecht, Netherlands, 1989. [14] Jian Liu, A weighted geometric inequality and its applications, Jour- nal of inequalities in pure and applied mathematics, Vol.9, Iss.2, Ar.58, 2008. [15] József Sándor, On the geometry of equilateral triangles, Forum Ge- ometricorum, Vol.5 (2005) 107–117. www.MATHVN.com - HOANG NGOC QUANG, Yen Bai

Các file đính kèm theo tài liệu này:

  • pdfkb5101.pdf
Luận văn liên quan